LSAT and Law School Admissions Forum

Get expert LSAT preparation and law school admissions advice from PowerScore Test Preparation.

 Administrator
PowerScore Staff
  • PowerScore Staff
  • Posts: 8916
  • Joined: Feb 02, 2011
|
#61085
Please post your questions below!
 Lsat180Please
  • Posts: 44
  • Joined: Sep 12, 2018
|
#61201
Hi! I would really appreciate a breakdown of this question. I found that I had a solid pre phrase coming off the stim and then the answer choices were so far off anything. I was stuck between C and E and would appreciate help with why C is wrong and E is correct. I thought C could work because using the negation technique, if some athletes who use PED's in cases in which they do not help... couldn't that mean side effects/ risks even if administered by a doctor. With E, i was confused because I thought, so what if there is a big competitive advantage to using PED's at unsafe levels because if they are administered by doctors the doctors monitor the doses, not the athletes and thus that takes the risk away. Thanks!
 Claire Horan
PowerScore Staff
  • PowerScore Staff
  • Posts: 408
  • Joined: Apr 18, 2016
|
#61250
Hi LSAT180Please,

Do you mind sharing what your strong prephrase was? It might help see if you were on the right track. :)

The conclusion is: "PEDs should be allowed, but only if administered under a doctor’s care to make sure they are taken only in safe doses."

We can prove that (E) is correct by using the assumption negation technique. It negates to: "Using PEDs at unsafe levels creates a big competitive advantage over using them at safe levels." Does this attack the conclusion? It may seem like it doesn't directly attack the conclusion, but (E) tends to undermine the control-but-don't-ban solution because it attacks the premise that, if PEDs were handled this way, health risks would disappear. (E) suggests there would still be health risks because the big competitive advantages would still incentivize athletes to take PEDs when not under a doctor's care. Don't forget that one of the premises says top athletes will do whatever it takes (which presumably includes breaking the rules).

Now let's try the assumption negation technique on answer choice (C). It negates to: Athletes take PEDs thinking they help performance in cases in which they do not help. That wouldn't attack the conclusion because if a doctor is administering them, they will be taken in safe doses and the health risks will disappear.

I hope this helps!
 Lsat180Please
  • Posts: 44
  • Joined: Sep 12, 2018
|
#61263
My prephrase for this question was something that says "just because it is administered by a doctor does not mean that there aren't potential risks" so I was looking for an answer that explained this. I am still a bit confused with E because I thought it was out of scope since his conclusion is specifically focusing on the facts that PED's are safe when under a doctors care. So if athletes are motivated to break the rules and take PEDs at unsafe levels, and thus outside the administration of the doctor, isn't that out of scope since he is specifically arguing that they are only safe under a doctors care (he never said they are safe outside of that). This "scope" part confused me because I saw the potential in E but I was a afraid it was out of scope! thanks!!
 Ben DiFabbio
PowerScore Staff
  • PowerScore Staff
  • Posts: 39
  • Joined: Aug 02, 2018
|
#61270
Lsat180Please wrote:My prephrase for this question was something that says "just because it is administered by a doctor does not mean that there aren't potential risks" so I was looking for an answer that explained this. I am still a bit confused with E because I thought it was out of scope since his conclusion is specifically focusing on the facts that PED's are safe when under a doctors care. So if athletes are motivated to break the rules and take PEDs at unsafe levels, and thus outside the administration of the doctor, isn't that out of scope since he is specifically arguing that they are only safe under a doctors care (he never said they are safe outside of that). This "scope" part confused me because I saw the potential in E but I was a afraid it was out of scope! thanks!!
Hey there,

In order to test whether answer choice (E) is out of scope, let's focus in on the conclusion, which is, in substance: The health risks of PEDs will disappear if *a rule is instituted* that PEDs can only be administered in safe doses under doctor's supervision.

The problem this proposal is purporting to solve is that banning PEDs altogether would be ineffective because athletes will just violate the rules if it provides a competitive advantage.

The flaw: What's keeping the athletes from violating this new, even more permissive rule and using PEDs in unsafe doses without a doctor's supervision? If an all-out ban wouldn't stop them from using PEDs in unsafe doses, then why would a slightly more limited control of PEDs be any more likely to stop them from using PEDs in unsafe doses?

Answer choice (E) is very much relevant to the flawed reasoning presented in the stimulus, since it exposes the unwarranted assumption that the athletes would abide by the new rule of taking PEDs only under a doctor's supervision, even while they would be expected to violate an outright ban. In order for the conclusion to follow, we would have to know that athletes wouldn't have an incentive to violate the new rule. So we have to assume that athletes wouldn't get a competitive advantage from using PEDs at unsafe levels.

I hope that helps!

Ben
 stu(dying)
  • Posts: 7
  • Joined: Aug 18, 2019
|
#67378
Hi, my reasoning for E is that if, PEDs at unsafe levels do create a competitive advantage over using them at safe levels then what's there to stop athletes from continuing to use them at unsafe levels?

Having doctors administer PEDs at safe levels would not solve the issue of athlete's using PEDs at unsafe levels. All the athletes really care about is using PEDs for competitive advantage and if using them at safe levels doesn't give them that, then screw the potential health risks!
 Jeremy Press
PowerScore Staff
  • PowerScore Staff
  • Posts: 1000
  • Joined: Jun 12, 2017
|
#67388
Hi stu(dying),

You've got it precisely! To echo Ben's very helpful discussion of the flaw in the reasoning of this stimulus, which you also agree with:
Ben DiFabbio wrote:
The flaw: What's keeping the athletes from violating this new, even more permissive rule and using PEDs in unsafe doses without a doctor's supervision? If an all-out ban wouldn't stop them from using PEDs in unsafe doses, then why would a slightly more limited control of PEDs be any more likely to stop them from using PEDs in unsafe doses?

Answer choice (E) is very much relevant to the flawed reasoning presented in the stimulus, since it exposes the unwarranted assumption that the athletes would abide by the new rule of taking PEDs only under a doctor's supervision, even while they would be expected to violate an outright ban. In order for the conclusion to follow, we would have to know that athletes wouldn't have an incentive to violate the new rule. So we have to assume that athletes wouldn't get a competitive advantage from using PEDs at unsafe levels.
Great work!

Jeremy
 frk215
  • Posts: 33
  • Joined: Sep 07, 2020
|
#83151
Hello!

I completely understand why E is correct, but I was hoping for further guidance on exactly why D is wrong.

I negated D as "athletes currently using PEDs can find doctors willing to prescribe them"

Taking into account that the main conclusion is "PEDs should be allowed but only if administered under a doctor's care to make sure they are taken only in safe doses." I figured that hey if this was already an option and thus athletes could've been doing under medical supervision anyways and this whole argument was still necessary [i.e. there were still health risks— I inferred this from the fact that there has to be health risks for them to be having a conversation about how to get rid of health risks] then the idea no longer really works. It can be handled by doctors and still have all the same health risks.

I know that I'm wrong but I want to know EXACTLY where I'm going wrong so I can call out such flaws in reasoning in the future. Thank you in advance!
User avatar
 KelseyWoods
PowerScore Staff
  • PowerScore Staff
  • Posts: 1079
  • Joined: Jun 26, 2013
|
#83794
Hi frk215!

You negated answer choice (D) correctly. But, as you know, the negation doesn't actually attack the argument. Let's talk about why:

The conclusion is that PEDs should be allowed, but only if administered under a doctor’s care to eliminate health risks.

The negation of answer choice (D) would be "Athletes currently using PEDs can find doctors willing to prescribe them."

But even if athletes currently using PEDs can find doctors willing to prescribe them, does that mean that all of them do? The conclusion is not that there aren't any athletes using PEDs under a doctor's care, it's that a doctor's care should be a requirement for their use. Thus, the negation of answer choice (D) doesn't attack the argument that doctors should be required for PEDs. Being able to find a doctor to prescribe them isn't the same as being required to do so.

Hope this helps!

Best,
Kelsey
User avatar
 crispycrispr
  • Posts: 71
  • Joined: Apr 08, 2021
|
#89265
@LSAT180 I think you've got the premise and conclusion backwards...I was confused by this question the first time I did it because of the same reason. If health risks will disappear when handled in this fashion is the premise, I think you're right that (E) is irrelevant. But the last sentence is a premise. It might help if you think about this in the context of a more relevant subject matter. Ignore the performance-enhancing part. Imagine if it's just drugs or shrooms. Naturally, someone advocating for the legalization of drugs would support their position by saying that if a doctor prescribes it, it'd be safe (health risks will disappear). So then the question is, even if health risks will disappear if a doctor prescribes it, will someone take the drug if it's only at the dosage prescribed by the doctor? Which would lead you to (E).

Get the most out of your LSAT Prep Plus subscription.

Analyze and track your performance with our Testing and Analytics Package.